(a) Compute the income increase or decrease from eliminating this segment. (b) Should the segment be continued or eliminated?

Survey of Accounting (Accounting I)
8th Edition
ISBN:9781305961883
Author:Carl Warren
Publisher:Carl Warren
Chapter12: Differential Analysis And Product Pricing
Section: Chapter Questions
Problem 1SEQ: Mario Company is considering discontinuing a product. The costs of the product consist of $20,000...
icon
Related questions
Question
None
Exercise 23-8 (Algo) Segment elimination LO P4
Marin Company makes several products, including canoes. The company reports a loss from its canoe segment (see below). All its
variable costs are avoidable, and $322,500 of its fixed costs are avoidable.
Segment Income (Loss)
Sales
Variable costs
Contribution margin
Fixed costs
Income (loss)
$ 1,068,200
763,000
305,200
367,000
$ (61,800)
(a) Compute the income increase or decrease from eliminating this segment.
(b) Should the segment be continued or eliminated?
Answer is complete but not entirely correct.
Complete this question by entering your answers in the tabs below.
Required A Required B
Compute the income increase or decrease from eliminating this segment.
Segment Elimination
Continue
Eliminate
Income Increase
(Decrease)
Analysis
Sales
$ 1,068,200 S 322,500 X
Variable costs
✓
763,000
322,500X
Contribution margin
305,200
0
Fixed costs
367,000
322,500 X
Income (loss)
$ (61,800)
$ (322,500)
$
(260,700)
Required A
Required B>
Transcribed Image Text:Exercise 23-8 (Algo) Segment elimination LO P4 Marin Company makes several products, including canoes. The company reports a loss from its canoe segment (see below). All its variable costs are avoidable, and $322,500 of its fixed costs are avoidable. Segment Income (Loss) Sales Variable costs Contribution margin Fixed costs Income (loss) $ 1,068,200 763,000 305,200 367,000 $ (61,800) (a) Compute the income increase or decrease from eliminating this segment. (b) Should the segment be continued or eliminated? Answer is complete but not entirely correct. Complete this question by entering your answers in the tabs below. Required A Required B Compute the income increase or decrease from eliminating this segment. Segment Elimination Continue Eliminate Income Increase (Decrease) Analysis Sales $ 1,068,200 S 322,500 X Variable costs ✓ 763,000 322,500X Contribution margin 305,200 0 Fixed costs 367,000 322,500 X Income (loss) $ (61,800) $ (322,500) $ (260,700) Required A Required B>
Expert Solution
steps

Step by step

Solved in 3 steps with 1 images

Blurred answer
Similar questions
  • SEE MORE QUESTIONS
Recommended textbooks for you
Survey of Accounting (Accounting I)
Survey of Accounting (Accounting I)
Accounting
ISBN:
9781305961883
Author:
Carl Warren
Publisher:
Cengage Learning